Đề bài: Cho tứ giác ABCD nội tiếp (O). AB cắt CD tại M, AD cắt BC tại N. AC cắt BD tại P. K là trung điểm MN, PK cắt (O) tại H. MH, NH cắt (O) tại I, J. Chứng minh KP chia đôi IJ.
Hướng dẫn:
Gọi T là điểm Miquel, G là giao điểm của NI và MJ.
Ta có $MI.MH=MA.MB=MN.MT$ suy ra tứ giác NTIH nội tiếp
Tương tự được tứ giác $MTHJ$ nội tiếp
Xét tam giác GMN có INHT và HTJM nội tiếp nên theo định lý Miquel GIHJ nội tiếp hay G thuộc (O).
Theo định lý Brocard ta có:
$OP.OT=R^2$ nên IJ giao GH tại P. Ta cũng có G, H, K thẳng hàng từ đó suy ra GK chia đôi IJ tại P.
Cách 2: Qua P kẻ đường thẳng vuông góc OP cắt (O) tại I, J như hình vẽ, Gọi H là giao điểm MI và (O), Ta sẽ chứng minh N,H, J thẳng hàng và P, H, K thẳng hàng.
Áp dụng định lý Pascal cho 6 điểm HIADCJ Ta có M, X, $N_1$ thẳng hàng ($N_1 $ là giao của AD và JH) $\Rightarrow AD \cap MX \equiv N_1$ nên N trùng $N_1$ hay N, H, J thẳng hàng.
Mặt khác theo định lý Brocard thì OP vuông MN nên IJ song song MN. Lại do P là trung điểm IJ nên P, H, K thẳng hàng. Như vậy ta có đpcm
Blog này tổng hợp các bài toán hay, các bài giảng chọn lọc về nhiều chủ đề: đại số, hình học, giải tích, số học và tổ hợp liên quan đến Toán Olympic và Toán thi ĐH.
Hiển thị các bài đăng có nhãn định lý Miquel. Hiển thị tất cả bài đăng
Hiển thị các bài đăng có nhãn định lý Miquel. Hiển thị tất cả bài đăng
Thứ Năm, 1 tháng 12, 2016
Thứ Ba, 25 tháng 10, 2016
Đường kính Brocard và tam giác đều " thủy túc "
Bài toán 1: Cho tam giác ABC nội tiếp (O), và K là điểm Lemoine có J, J' là hai điểm đẳng động . Nếu OK cắt (O) tại Q và R thì (QRJJ')=-1
Chứng minh:
Ta đã chứng minh O,K, J, J' thẳng hàng và do J và J' nghịch đảo nhau đối với (O) nên (QRJJ')=-1
QR được gọi là đường kính Brocard của tam giác ABC
Bài toán 2:
Chứng minh rằng có đúng hai điểm đối với một tam giác sao cho chân đường vuông góc hạ
từ chúng đến ba cạnh của tam giác tạo thành một tam giác đều
Chứng minh:
Theo bài toán 1 điểm có hình chiếu lên 3 cạnh tam giác tạo thành 1 tam giác cân nằm
trên một đường tròn apollonius.
Để có tam gíac đều thì điểm đó phải nằm trên 3 đường tròn apollonius của tam giác tức
là hai điểm đẳng động J, J'
của tam giác
Chú ý : Tam giác đều tạo bởi 3 hình chiếu của điểm J lên ba cạnh của tam giác thường
gọi là tam giác đều " thủy túc " của điểm J
Bài toán 3: Trong tất cả các tam giác đều có đỉnh nằm trên ba cạnh của một tam giác thì tam giác đều
thủy túc của điểm đẳng động thứ nhất J của tam giác có diện tích nhỏ nhất.
Chứng minh:
Nhắc lại định lý Miquel :
Cho tam giác ABC và ba điểm L, M, N nằm trên BC, CA, AB. Khi đó ba đường tròn
(AMN),(BLN),(CLM) đồng quy
Gọi các điểm như trong hình vẽ
Ta có:
$\widehat{JLM}=\widehat{JCM}=\widehat{JL'M} \\ \widehat{JLN}=\widehat{JBN}=\widehat{JL'N'} $
Cộng lại ta được:
$\widehat{MLN}=\widehat{M'L'N'}=60^o$
Phép đồng dạng ( vị tự quay) tâm J với tỷ số $r=\frac{JL'}{JL} \le 1, \alpha=\widehat{LJL'}$
biến tam giác
LMN thành tam giác L'M'N'. Theo bài toán 3 suy ra J là điểm đẳng động thứ nhất.
Chứng minh:
Ta đã chứng minh O,K, J, J' thẳng hàng và do J và J' nghịch đảo nhau đối với (O) nên (QRJJ')=-1
QR được gọi là đường kính Brocard của tam giác ABC
Bài toán 2:
Chứng minh rằng có đúng hai điểm đối với một tam giác sao cho chân đường vuông góc hạ
từ chúng đến ba cạnh của tam giác tạo thành một tam giác đều
Chứng minh:
Theo bài toán 1 điểm có hình chiếu lên 3 cạnh tam giác tạo thành 1 tam giác cân nằm
trên một đường tròn apollonius.
Để có tam gíac đều thì điểm đó phải nằm trên 3 đường tròn apollonius của tam giác tức
là hai điểm đẳng động J, J'
của tam giác
Chú ý : Tam giác đều tạo bởi 3 hình chiếu của điểm J lên ba cạnh của tam giác thường
gọi là tam giác đều " thủy túc " của điểm J
Bài toán 3: Trong tất cả các tam giác đều có đỉnh nằm trên ba cạnh của một tam giác thì tam giác đều
thủy túc của điểm đẳng động thứ nhất J của tam giác có diện tích nhỏ nhất.
Chứng minh:
Nhắc lại định lý Miquel :
Cho tam giác ABC và ba điểm L, M, N nằm trên BC, CA, AB. Khi đó ba đường tròn
(AMN),(BLN),(CLM) đồng quy
Ta có:
$\widehat{JLM}=\widehat{JCM}=\widehat{JL'M} \\ \widehat{JLN}=\widehat{JBN}=\widehat{JL'N'} $
Cộng lại ta được:
$\widehat{MLN}=\widehat{M'L'N'}=60^o$
Phép đồng dạng ( vị tự quay) tâm J với tỷ số $r=\frac{JL'}{JL} \le 1, \alpha=\widehat{LJL'}$
biến tam giác
LMN thành tam giác L'M'N'. Theo bài toán 3 suy ra J là điểm đẳng động thứ nhất.
Nhãn:
điểm đẳng động,
điểm Fermat,
điểm Lemoine,
định lý Miquel,
đường kính Brocard,
đường tròn Apollonius,
hàng điểm điều hoà,
hình học,
tam giác đều thủy túc
Thứ Hai, 11 tháng 7, 2016
Kết hợp giữa phép nghịch đảo và định lý Miquel
Bài 1: Cho tam giác ABC, đường tròn (K) qua B, C cắt AC, AB tại E, F. BE cắt CF tại H. Gọi Q là tâm đường tròn (HEF), L là tâm đường tròn KBC. Chứng minh rằng Q, K, L thẳng hàng.
Lời giải
Gọi S là giao của (ABE) và (ACF) theo định lý Miquel ta có S thuộc (BFH) và (HEC). Suy ra:
∠BSC=360°-∠BSH-∠HSC=∠BFC+∠BEC=∠BKC Suy ra tứ giác BSKC nội tiếp.
Xét phép nghịch đảo tâm A đối với (K), biến:
F thành B
E thành C
Suy ra EB thành (ABE), CF thành (ACF)
Nên biến H là giao điểm EB và FC thành S là giao điểm (ABE), (ACF)
Nên phép vị tự này biến (HEF) thành (BSC) nên tồn tại phép vị tự tâm A tỉ số k biến Q thành L.
Vậy A, Q, L thẳng hàng
Bài 2: (tiếng anh) Let $\triangle ABC$. A circle passes through $B,C$ intersects $AC,AB$ at $E,F$. The lines passes through $E,F$ and perpendicular to $AC,AB$ intersect together at $O. M,N$ lies on $EO,FO$,respectively. Draw $CQ\perp AM,BP\perp AN. BP$ and $AM$ intersects together at $I.MB$ intersect $NC$ at $K$.
Prove that $O,I,K$ are collinear.
Solution
Lemma: Let $ABCD$ is a quadrilateral. $AB$ cuts $CD$ at $G$; $AD$ cuts $BC$ at $K$. Then three circles with diameter $AC,BD,GK$ have the same radical axis.
Proof
Let $H,H'$ are the orthocenters of $\triangle GAD$ and $\triangle KCD$ respectively. Suppose that $GH,AH,DH$ cut $AD,GD,GA$ at $P,Q,R$ respectively. We have $HG.HP=HA.HQ=HD.HR$ so $H$ is on the radical axis of three circles with diameter $AC,BD,GK$. Similarly, we have $H'$ is on the radical axis of three circles with diameter $AC,BD,GK$, too. Hence, $HH'$ is the radical axis of three circles with diameter $AC,BD,GK$ so three cicles with diameter $AC,BD,GK$ have the same radical axis.
Back to this problem Let $X,Y,Z$ are the midpoints of $AO,AI,AK$ respectively. $BC$ cuts $MN$ at $G$. $H$ lies on $AK$ such that $GH \perp AK$. We have $AF.AB=AE.AC$ so $A$ lies on the radical axis of the circles with diameter $BN,CM$. According this lemma, we have $AH.AK=AF.AB=AE.AC$. The inversion center $A$, radius $AF.AB$ : $(X;XA) \mapsto BC; (Y;YA) \mapsto MN; (Z,ZA) \mapsto$ the line passing $H$ and perpendicular $AK$. But $BC, MN$ and the line passing $H$ and perpendicular $AK$ are concurrent at $G$. So $(X;XA), (Y;YA), (Z;ZA)$ have the same radical axis $\Longrightarrow$ $X,Y,Z$ are collinear. Hence $O,I,K$ are collinear.
Bài 3: Cho tam giác ABC. Đường tròn (O) đi qua B, C cắt AC, AB tại E, F. BE cắt CF tại D. H là hình chiếu của O trên AD. K, L là tâm (AFC) và (AEB). I là giao điểm khác H của (KHF) và (LHE). CMR: AI đi qua trung điểm BC.
Lời giải (Huỳnh Bách Khoa)
Lưu ý thêm:
Ta cần bổ đề sau: Gọi H là giao của (BDF) và (DEC). Thì OH vuông DH. Ta có thể gọi M, N là trung điểm FC, EB. Sau đó dùng vị tự quay để suy ra OHMND đồng viên đường tròn đường kính OD. (China 1992)
Và một số sai sót: M thuộc (LEH) do ∠MLH=∠HEC
Phép gọi T là đối xứng D qua trung điểm BC thường hay gặp để tạo ra đường đẳng giác góc A với AH
Và còn ∠DHN=∠NAH=∠DXN
Về phần phép nghịch đảo là phương tích AE.AC, biến E thành C, F thành B. Thay điểm G thành D. Do I là giao của (MEH) và (NHF) nên biến thành Z là giao của (BDX) và (CDY)
Lời giải
∠BSC=360°-∠BSH-∠HSC=∠BFC+∠BEC=∠BKC Suy ra tứ giác BSKC nội tiếp.
Xét phép nghịch đảo tâm A đối với (K), biến:
F thành B
E thành C
Suy ra EB thành (ABE), CF thành (ACF)
Nên biến H là giao điểm EB và FC thành S là giao điểm (ABE), (ACF)
Nên phép vị tự này biến (HEF) thành (BSC) nên tồn tại phép vị tự tâm A tỉ số k biến Q thành L.
Vậy A, Q, L thẳng hàng
Bài 2: (tiếng anh) Let $\triangle ABC$. A circle passes through $B,C$ intersects $AC,AB$ at $E,F$. The lines passes through $E,F$ and perpendicular to $AC,AB$ intersect together at $O. M,N$ lies on $EO,FO$,respectively. Draw $CQ\perp AM,BP\perp AN. BP$ and $AM$ intersects together at $I.MB$ intersect $NC$ at $K$.
Prove that $O,I,K$ are collinear.
Solution
Lemma: Let $ABCD$ is a quadrilateral. $AB$ cuts $CD$ at $G$; $AD$ cuts $BC$ at $K$. Then three circles with diameter $AC,BD,GK$ have the same radical axis.
Proof
Let $H,H'$ are the orthocenters of $\triangle GAD$ and $\triangle KCD$ respectively. Suppose that $GH,AH,DH$ cut $AD,GD,GA$ at $P,Q,R$ respectively. We have $HG.HP=HA.HQ=HD.HR$ so $H$ is on the radical axis of three circles with diameter $AC,BD,GK$. Similarly, we have $H'$ is on the radical axis of three circles with diameter $AC,BD,GK$, too. Hence, $HH'$ is the radical axis of three circles with diameter $AC,BD,GK$ so three cicles with diameter $AC,BD,GK$ have the same radical axis.
Back to this problem Let $X,Y,Z$ are the midpoints of $AO,AI,AK$ respectively. $BC$ cuts $MN$ at $G$. $H$ lies on $AK$ such that $GH \perp AK$. We have $AF.AB=AE.AC$ so $A$ lies on the radical axis of the circles with diameter $BN,CM$. According this lemma, we have $AH.AK=AF.AB=AE.AC$. The inversion center $A$, radius $AF.AB$ : $(X;XA) \mapsto BC; (Y;YA) \mapsto MN; (Z,ZA) \mapsto$ the line passing $H$ and perpendicular $AK$. But $BC, MN$ and the line passing $H$ and perpendicular $AK$ are concurrent at $G$. So $(X;XA), (Y;YA), (Z;ZA)$ have the same radical axis $\Longrightarrow$ $X,Y,Z$ are collinear. Hence $O,I,K$ are collinear.
Bài 3: Cho tam giác ABC. Đường tròn (O) đi qua B, C cắt AC, AB tại E, F. BE cắt CF tại D. H là hình chiếu của O trên AD. K, L là tâm (AFC) và (AEB). I là giao điểm khác H của (KHF) và (LHE). CMR: AI đi qua trung điểm BC.
Lời giải (Huỳnh Bách Khoa)
Lưu ý thêm:
Ta cần bổ đề sau: Gọi H là giao của (BDF) và (DEC). Thì OH vuông DH. Ta có thể gọi M, N là trung điểm FC, EB. Sau đó dùng vị tự quay để suy ra OHMND đồng viên đường tròn đường kính OD. (China 1992)
Và một số sai sót: M thuộc (LEH) do ∠MLH=∠HEC
Phép gọi T là đối xứng D qua trung điểm BC thường hay gặp để tạo ra đường đẳng giác góc A với AH
Và còn ∠DHN=∠NAH=∠DXN
Về phần phép nghịch đảo là phương tích AE.AC, biến E thành C, F thành B. Thay điểm G thành D. Do I là giao của (MEH) và (NHF) nên biến thành Z là giao của (BDX) và (CDY)
Thứ Sáu, 24 tháng 6, 2016
Điều kiện cần và đủ của tứ giác có hai đường chéo vuông góc trong IMO shorlist 2008
Bài: Cho tứ giác lồi ABCD. Chứng minh rằng tồn tại điểm P nằm trong tứ giác thỏa mãn:
$\widehat{PAB}+\widehat{PDC}=\widehat{PBC}+\widehat{PAD}=\widehat{PCD}+\widehat{PBA}=\widehat{PDA}+\widehat{PCB}=90^o$
Khi và chỉ khi hai đường chéo AC và BD vuông góc.
Lời giải:
Giả sử tồn tại P thỏa mãn đề bài. Gọi M là điểm Miquel của tứ giác toàn phần tạo bởi các đường thẳng AB, BC, CD, DA, R là giao BC với DA
Theo điều kiện đề bài:
$180^o=\widehat{PAB}+\widehat{PDC}+\widehat{PCD}+\widehat{PBA}=360-\widehat{BPA}-\widehat{CPD} \Rightarrow \widehat{BPA}=180^o-\widehat{DPC}$
Mặt khác: $\widehat{CRD}=180^o-\widehat{RDC}-\widehat{RCD}=90^o-\widehat{PDC}-\widehat{PCD}=\widehat{CPD}-90^o \Rightarrow \widehat{ARB}=90^o-\widehat{BPA}$
Tương tự ta cũng có: $\widehat{BQC}=90^o-\widehat{BPC}$
Vậy: $\widehat{CPA}=\widehat{BPA}+\widehat{BPC}=180^o-\widehat{BRA}-\widehat{BQC}=180^o-\widehat{BMA}-\widehat{CMB}=180^o-\widehat{CMA}$
Suy ra tứ giác APCM nội tiếp, tương tự DPBM nội tiếp.
$\widehat{BCA}+\widehat{DBC}=\widehat{PCB}-\widehat{PCA}+\widehat{PBC}+\widehat{PBD}=180^o-\widehat{CPB}+\widehat{PMD}-\widehat{DMA}=180^o-\widehat{CPB}-\widehat{AMD}=180^o-\widehat{CPB}-\widehat{DQA}=90^o$
Vậy $AC \perp BD$
Chiều ngược lại chỉ cần chứng minh tồn tại điểm liên hợp đẳng giác với điểm J giao điểm hai đường chéo, trong tứ giác ABCD. Theo lời giải của bạn Huỳnh Bách Khoa
$\widehat{PAB}+\widehat{PDC}=\widehat{PBC}+\widehat{PAD}=\widehat{PCD}+\widehat{PBA}=\widehat{PDA}+\widehat{PCB}=90^o$
Khi và chỉ khi hai đường chéo AC và BD vuông góc.
Lời giải:
Theo điều kiện đề bài:
$180^o=\widehat{PAB}+\widehat{PDC}+\widehat{PCD}+\widehat{PBA}=360-\widehat{BPA}-\widehat{CPD} \Rightarrow \widehat{BPA}=180^o-\widehat{DPC}$
Mặt khác: $\widehat{CRD}=180^o-\widehat{RDC}-\widehat{RCD}=90^o-\widehat{PDC}-\widehat{PCD}=\widehat{CPD}-90^o \Rightarrow \widehat{ARB}=90^o-\widehat{BPA}$
Tương tự ta cũng có: $\widehat{BQC}=90^o-\widehat{BPC}$
Vậy: $\widehat{CPA}=\widehat{BPA}+\widehat{BPC}=180^o-\widehat{BRA}-\widehat{BQC}=180^o-\widehat{BMA}-\widehat{CMB}=180^o-\widehat{CMA}$
Suy ra tứ giác APCM nội tiếp, tương tự DPBM nội tiếp.
$\widehat{BCA}+\widehat{DBC}=\widehat{PCB}-\widehat{PCA}+\widehat{PBC}+\widehat{PBD}=180^o-\widehat{CPB}+\widehat{PMD}-\widehat{DMA}=180^o-\widehat{CPB}-\widehat{AMD}=180^o-\widehat{CPB}-\widehat{DQA}=90^o$
Vậy $AC \perp BD$
Chiều ngược lại chỉ cần chứng minh tồn tại điểm liên hợp đẳng giác với điểm J giao điểm hai đường chéo, trong tứ giác ABCD. Theo lời giải của bạn Huỳnh Bách Khoa
Thứ Năm, 23 tháng 6, 2016
Dùng định lý Miquel để giải bài toán hình học
Bài 1: Cho tam giác ABC nội tiếp (O). Các tiếp tuyến tại B, C cắt nhau tại T. Đường thẳng qua A vuông góc AT cắt BC tại S. $B_1, C_1$ trên ST (T nằm giữa $B_1$ và $C_1$, $B_1 $nằm giữa S và T) sao cho $B_1T=BT=C_1T$. Chứng minh rằng tam giác $ABC$ đồng dạng tam giác $AB_1C_1$.
Lời giải:
Gọi M là trung điểm BC thì T,M,A,S đồng viên.
Ta cũng có $B_1,C_1,B,C$ đồng viên
Gọi K là giao của $BB_1$ và $CC_1$ thì $\widehat{BKC}=180^o-\widehat{KBC}-\widehat{BCK}=180^o-\widehat{KBC}-\widehat{B_1BT}=\widehat{TBC}$
Tương tự ta suy ra BT và CT là tiếp tuyến của $(KBC)$
Suy ra K thuộc (ABC) (Vì tâm của KBC là giao của đường thẳng qua B và C vuông BT và CT).
Vậy A là giao của (KBC) và (SMT). Gọi J là giao của $CB_1$ và $BC_1$ thì theo định lý Brocard $TJ \perp SSK$ tại A', Theo Pascal đảo cho 6 điểm BBKCJ ta suy ra J thuộc (KBC), mà A' lại thuộc (KJC) (Do IJ vuông SK tại A') Suy ra A' là giao của (KBC) và (SMT) vậy $A' \equiv A$
Hoặc cách khác: Do TA đã vuông SA, nên ta phải chứng minh TA vuông AK ( điều này có thể chứng minh bằng biến đổi góc cho S,A,K thẳng hàng).
Từ đó B là điểm Miquel của tam giác KSC nên tứ giác $ASB_1B$ nội tiếp, Suy ra A là điểm Miquel của $BCC_1B_1SK$. Cuối cùng theo phép vị tự quay tâm A góc quay $\varphi$ tỉ số $k$ ta có diều phải cm.
Lời giải:
Gọi M là trung điểm BC thì T,M,A,S đồng viên.
Ta cũng có $B_1,C_1,B,C$ đồng viên
Gọi K là giao của $BB_1$ và $CC_1$ thì $\widehat{BKC}=180^o-\widehat{KBC}-\widehat{BCK}=180^o-\widehat{KBC}-\widehat{B_1BT}=\widehat{TBC}$
Tương tự ta suy ra BT và CT là tiếp tuyến của $(KBC)$
Suy ra K thuộc (ABC) (Vì tâm của KBC là giao của đường thẳng qua B và C vuông BT và CT).
Vậy A là giao của (KBC) và (SMT). Gọi J là giao của $CB_1$ và $BC_1$ thì theo định lý Brocard $TJ \perp SSK$ tại A', Theo Pascal đảo cho 6 điểm BBKCJ ta suy ra J thuộc (KBC), mà A' lại thuộc (KJC) (Do IJ vuông SK tại A') Suy ra A' là giao của (KBC) và (SMT) vậy $A' \equiv A$
Hoặc cách khác: Do TA đã vuông SA, nên ta phải chứng minh TA vuông AK ( điều này có thể chứng minh bằng biến đổi góc cho S,A,K thẳng hàng).
Từ đó B là điểm Miquel của tam giác KSC nên tứ giác $ASB_1B$ nội tiếp, Suy ra A là điểm Miquel của $BCC_1B_1SK$. Cuối cùng theo phép vị tự quay tâm A góc quay $\varphi$ tỉ số $k$ ta có diều phải cm.
Mở rộng của bài IMO 2005
Đề bài: Cho tứ giác ABCD có DA, BC không song song, P là giao điểm của các đường chéo AC, BD. M, N chạy trên DA, BC sao cho $\dfrac{DM}{DA}=\dfrac{BN}{BC}$. MN theo thứ tự cắt AC, BD tại Q, R. Chứng minh rằng đường tròn (PQR) đi qua một điểm cố định khác P.
Lời giải:
Gọi O là giao điểm của (PAD) và (PBC) thì khi đó:
Xét phép vị tự quay:
$S(O,k, \varphi ):D\rightarrow B\\: A\rightarrow C\\:\Rightarrow DA\rightarrow BC\\:M\rightarrow N(Do:\frac{DM}{AM}=\frac{BN}{NV})$
Suy ra tam giác OAC đồng dạng tam giác OMN, suy ra $\widehat{MNO}=\widehat{QCO}$ Suy ra tứ giác ONQC nội tiếp. Nên O là điểm Miquel của tứ giác toàn phần BPCQNR. Hay O $\in (PQR)$
Lời giải:
Gọi O là giao điểm của (PAD) và (PBC) thì khi đó:
Xét phép vị tự quay:
$S(O,k, \varphi ):D\rightarrow B\\: A\rightarrow C\\:\Rightarrow DA\rightarrow BC\\:M\rightarrow N(Do:\frac{DM}{AM}=\frac{BN}{NV})$
Suy ra tam giác OAC đồng dạng tam giác OMN, suy ra $\widehat{MNO}=\widehat{QCO}$ Suy ra tứ giác ONQC nội tiếp. Nên O là điểm Miquel của tứ giác toàn phần BPCQNR. Hay O $\in (PQR)$
Thứ Năm, 28 tháng 4, 2016
Hai bài toán hình học nổi tiếng trong kì thi IMO
Đây là hai bài hình học trong kì thi toán Quốc tế được thầy Trần Quang Hùng đánh giá là đặc biệt và hay trong 7 bài hình học mà thầy đăng
Bài toán 1 (IMO 1985 bài 5 ngày 2). Đường tròn với tâm O đi qua các đỉnh A và C của tam giác ABC cắt các đoạn thẳng BA,BC lần thứ hai tại các điểm K và N. Gọi M là giao điểm của các đường tròn ngoại tiếp tam giác ABC và KBN (khác B). Chứng minh rằng $∠OMB=90^o$.
Bài toán 2 (IMO 1996 bài 2 ngày 1). Cho điểm P nằm trong tam giác ABC sao cho ∠APB−∠ACB=∠APC−∠ABC. Gọi D,E lần lượt là tâm nội tiếp các tam giác APB,APC. Chứng minh rằng AP,BD,CE đồng quy.
Bài toán 1 (IMO 1985 bài 5 ngày 2). Đường tròn với tâm O đi qua các đỉnh A và C của tam giác ABC cắt các đoạn thẳng BA,BC lần thứ hai tại các điểm K và N. Gọi M là giao điểm của các đường tròn ngoại tiếp tam giác ABC và KBN (khác B). Chứng minh rằng $∠OMB=90^o$.
Bài toán 2 (IMO 1996 bài 2 ngày 1). Cho điểm P nằm trong tam giác ABC sao cho ∠APB−∠ACB=∠APC−∠ABC. Gọi D,E lần lượt là tâm nội tiếp các tam giác APB,APC. Chứng minh rằng AP,BD,CE đồng quy.
Tôi xin trình bày tắt lời giải của hai bài toán.
Bài toán 1: Gọi P là giao điểm các đường thẳng AC và KN thì 4 điểm M, P, A, K cùng thuộc một đường tròn (theo định lý Miquel) ngoài ra ta cũng có M nằm trên đoạn BP.
Theo phương tích ta có:
$BO^2-PO^2=BO^2-R^2-(PO^2-R^2)=\overline{BM}.\overline{BP}-\overline{PM}.\overline{PB}=BM^2-PM^2$
Theo định lí 4 điểm ta có điều phải chứng minh.
Bài toán 2: Ta sẽ sử dụng bổ đề: Cho P là một điểm năm trong tam giác ABC. X,Y,Z lần lượt là chân đường vuông góc hạ từ P xuống BC, CA, AB.
Khi đó:$ PA=\dfrac{YZ}{sin A}$ và $\angle APB-\angle ACB=\angle XZY$
Bổ đề này xin dành cho bạn đọc chứng minh.
Trở lại bài toán. Ta có: $\angle APB-\angle ACB=\angle XZY$ và $\angle APC-\angle ABC=\angle XYZ$
Suy ra tam giác XYZ cân, với XY=XZ. Từ đó:
$PCsinACB=PB.sin ABC$ dùng thêm định lí hàm sin ta được
$\dfrac{AB}{PB}=\dfrac{AC}{PC}$
Nếu BD cắt AP tại W thì theo định lí Stewwart ta có $\dfrac{AB}{PB}=\dfrac{AC}{PC}=\dfrac{AW}{PW}$ và CW cũng là phân giác, ta có điều phải chứng minh.
Đăng ký:
Bài đăng (Atom)
Bất đẳng thức tuyển sinh lớp 10 chọn lọc
Trong bài viết này, tác giả giới thiệu một số bài BĐT nhẹ nhàng nhưng ý tưởng tương đối mới, mức độ phù hợp với đề thi tuyển sinh vào lớp...
-
I) Hàm phần nguyên: 1) Định nghĩa Phần nguyên của một số thực x là số nguyên lớn nhất không vượt quá x. Kí hiệu là [x]. 2) Tính chất...
-
Định nghĩa 1: Một số nguyên a được gọi là thặng dư bình phương mod n nếu tồn tại số nguyên x sao cho $x^2 \equiv a (mod n)$ Ta cũng có th...
-
Trong thế giới bất đẳng thức , ngoài những bất đẳng thức kinh điển và được áp dụng rất nhiều như bất đẳng thức AM – GM, bất đẳng thức Cauc...